Difference between revisions of "2011 USAJMO Problems/Problem 5"

(Solution)
(Solution)
Line 4: Line 4:
  
 
==Solution==
 
==Solution==
It's well known that <math>ABCD</math> is a harmonic quadrilateral. Since <math>\overline{DE} \parallel \overline{AC}</math>, we have that <math>AD=CE\rightarrow \angle ABD=\angle CBE</math>. But <math>ABCD</math> is a harmonic quadrilateral, thus <math>\overline{BD}</math> is a symmedian of triangle <math>ABC</math>, from which it follows that <math>\overline{BE}</math> is a median of <math>\triangle ABC</math>. <math>\blacksquare</math>
+
Since <math>\overline{DE} \parallel \overline{AC}</math>, we have that <math>AD=CE\rightarrow \angle ABD=\angle CBE</math>. But it is well known that <math>ABCD</math> is a harmonic quadrilateral, thus <math>\overline{BD}</math> is a symmedian of triangle <math>ABC</math>, from which it follows that <math>\overline{BE}</math> is a median of <math>\triangle ABC</math>. <math>\blacksquare</math>
  
 
== Solution 1 ==
 
== Solution 1 ==

Revision as of 09:19, 2 October 2021

Problem

Points $A$, $B$, $C$, $D$, $E$ lie on a circle $\omega$ and point $P$ lies outside the circle. The given points are such that (i) lines $PB$ and $PD$ are tangent to $\omega$, (ii) $P$, $A$, $C$ are collinear, and (iii) $\overline{DE} \parallel \overline{AC}$. Prove that $\overline{BE}$ bisects $\overline{AC}$.

Solution

Since $\overline{DE} \parallel \overline{AC}$, we have that $AD=CE\rightarrow \angle ABD=\angle CBE$. But it is well known that $ABCD$ is a harmonic quadrilateral, thus $\overline{BD}$ is a symmedian of triangle $ABC$, from which it follows that $\overline{BE}$ is a median of $\triangle ABC$. $\blacksquare$

Solution 1

Connect segment PO, and name the interaction of PO and the circle as point M.

Since PB and PD are tangent to the circle, it's easy to see that M is the midpoint of arc BD.

∠ BOA = 1/2 arc AB + 1/2 arc CE

Since AC // DE, arc AD = arc CE,

thus, ∠ BOA = 1/2 arc AB + 1/2 arc AD = 1/2 arc BD = arc BM = ∠ BOM

Therefore, PBOM is cyclic, ∠ PFO = ∠ OBP = 90°, AF = AC (F is the interaction of BE and AC)

BE bisects AC, proof completed!

~ MVP Harry

Solution 2

Let $O$ be the center of the circle, and let $X$ be the intersection of $AC$ and $BE$. Let $\angle OPA$ be $x$ and $\angle OPD$ be $y$.

$\angle OPB = \angle OPD = y$, $\angle BED = \frac{\angle DOB}{2} = 90-y$, $\angle ODE = \angle PDE - 90 = 90-x-y$ $\angle OBE = \angle PBE - 90 = x = \angle OPA$

Thus $PBXO$ is a cyclic quadrilateral and $\angle OXP = \angle OBP = 90$ and so $X$ is the midpoint of chord $AC$.

~pandadude

Solution 3

This is the solution from EGMO Problem 1.43 page 242

Let $O$ be the center of the circle, and let $M$ be the midpoint of $AC$. Let $\theta$ denote the circle with diameter $OP$. Since $\angle OBP = \angle OMP = \angle ODP = 90^\circ$, $B$, $D$, and $M$ all lie on $\theta$.

[asy] import graph;  unitsize(2 cm);  pair A, B, C, D, E, M, O, P; path circ;  O = (0,0); circ = Circle(O,1); B = dir(100); D = dir(240); P = extension(B, B + rotate(90)*(B), D, D + rotate(90)*(D)); C = dir(-40); A = intersectionpoint((P--(P + 0.9*(C - P))),circ); E = intersectionpoint((D + 0.1*(C - A))--(D + C - A),circ); M = (A + C)/2;  draw(circ); draw(P--B); draw(P--D); draw(P--C); draw(B--E); draw(D--E); draw(O--B); draw(O--D); draw(O--M); draw(O--P); draw(Circle((O + P)/2, abs(O - P)/2),dashed); draw(D--M);  dot("$A$", A, NE); dot("$B$", B, NE); dot("$C$", C, SE); dot("$D$", D, S); dot("$E$", E, S); dot("$M$", M, NE); dot("$O$", O, dir(0)); dot("$P$", P, W); label("$\theta$", (O + P)/2 + abs(O - P)/2*dir(120), NW); [/asy]

Since quadrilateral $BOMP$ is cyclic, $\angle BMP = \angle BOP$. Triangles $BOP$ and $DOP$ are congruent, so $\angle BOP = \angle BOD/2 = \angle BED$, so $\angle BMP = \angle BED$. Because $AC$ and $DE$ are parallel, $M$ lies on $BE$ (using Euclid's Parallel Postulate).

Solution 4

Note that by Lemma 9.9 of EGMO, $(A,C;B,D)$ is a harmonic bundle. We project through $E$ onto $\overline{AC}$, \[-1=(A,C;B,D)\stackrel{E}{=}(A,C;M,P_{\infty})\] Where $P_{\infty}$ is the point at infinity for parallel lines $\overline{DE}$ and $\overline{AC}$. Thus, we get $\frac{MA}{MC}=-1$, and $M$ is the midpoint of $AC$. ~novus677

The problems on this page are copyrighted by the Mathematical Association of America's American Mathematics Competitions. AMC logo.png